If the set U = {all positive integers} and set A = {x|x ∈ U and x is an odd positive integer}, which describes the complement of set A, Ac?

Ac = {x|x ∈ U and is a negative integer}
Ac = {x|x ∈ U and is zero}
Ac = {x|x ∈ U and is not an integer}
Ac = {x|x ∈ U and is an even positive integer}

Answers

Answer 1

Answer: Choice D) set of even integers

=======================================================

Explanation:

U = universal set = {all positive integers} = {1, 2, 3, 4, 5, ...}

A = {stuff in set U that is odd} = {1, 3, 5, 7, 9, ...}

[tex]A^c[/tex] = {stuff in set U but NOT from set A} = {2, 4, 6, 8, ...}

[tex]A^c[/tex] = {set of even numbers from set U}

In set theory, the complement is simply the complete opposite. If the number 7 is found in set A for instance, then 7 is not going to live in the complement [tex]A^c[/tex]

The opposite of the odd numbers is the set of even numbers.

Therefore, we go for choice D as our final answer.

Side note: if we union set A with its complement [tex]A^c[/tex] then we get the universal set as a result. [tex]A \cup A^c = \text{universal set} = \text{set of all positive integers}[/tex]

Answer 2

The solution is Option D.

Ac =  {x|x ∈ U and is an even positive integer} where Ac represents the complement of set A

What is union and intersection of sets?

The union of two sets A and B is the set of all those elements which are either in A or in B, i.e. A ∪ B, whereas the intersection of two sets A and B is the set of all elements which are common. The intersection of these two sets is denoted by A ∩ B

The union of two sets is a new set that contains all of the elements that are in at least one of the two sets

The intersection of two sets is a new set that contains all of the elements that are in both sets

Given data ,

Let the set U be = {all positive integers}

So , the values of set U = { 1 , 2 , 3 , 4 , 5 .. }

And , let the set A = {x|x ∈ U and x is an odd positive integer}

So , the value of set A = { 1 , 3 , 5 , 7 , ... }

And , the complement of set  A is Ac

The value of set Ac is all the even positive odd integers

So , the value of set Ac = { 2 , 4 , 6 , 8 .. }

Therefore , the value of set Ac = { 2 , 4 , 6 , 8 .. }

Hence , the set Ac = {x|x ∈ U and is an even positive integer}

To learn more about sets click :

https://brainly.com/question/28278437

#SPJ2


Related Questions

How to find the point slope form with the slope of 3 and the point of -2/6

Answers

[tex]\quad \huge \quad \quad \boxed{ \tt \:Answer }[/tex]

[tex]\qquad \tt \rightarrow \:y-6 = 3(x + 2) [/tex]

____________________________________

[tex] \large \tt Solution \: : [/tex]

Equation of line in point - slope form :

[tex]\qquad \tt \rightarrow \:y-y_1 = m(x - x_1)[/tex]

[tex] \tt \:m = slope=3[/tex]

[tex]\tt y_1 = y \: \: coordinate \: \: of \: \: point = 6[/tex]

[tex] \tt \:x_1 = x \: \: coordinate \: \: of \: \: point = - 2[/tex]

Here :

[tex]\qquad \tt \rightarrow \:y - 6 = 3(x - ( - 2))[/tex]

[tex]\qquad \tt \rightarrow \:y - 6 = 3(x + 2)[/tex]

Simplify (3 + √5 )(2 + √5)
need the answer ASAP, WILL GIVE BRAINLIEST ​

Answers

Answer:

11 + 5√5

Step-by-step explanation:

6 + 3√5 +2√5 + 5

11 + 5√5

root n * root n = n , which is why there is a 5 at the end

Answer:

the answer is 11+5√5

Step-by-step explanation:

B/c (a+b).(c+d)=ac+ad+bc+bad(3+√5)(2+√5)=3.2+3.√5+√5.2+√5√5=6+3√5+2√5+√5√5=6+3√5+2√5+5=11+3√52√5=11+5√5

erved,
The graph of function fis shown on the coordinate plane. Graph the line representing function g, if g is defined as shown below.
g(x) = 2f(x-1)

Answers

Using the graph attached, If g(x) = 2f(x-1), the graph of the function will be:  g(x) = x - 7.

What is the function about?

Using the graph, the function is:

f(x-1) =1/2 (x-1) - 3

f(x-1) = 1/2x - 1/2 - 3/1

f(x-1) = 1/2x - 7/2

So:

g(x) = 2f(x-1)

g(x) = 2 (1/2x - 7/2)

g(x) = x - 7

Therefore,:

At  x = 0,  y = -7

y = 0, x = -7

Thus, Using the graph attached, If g(x) = 2f(x-1), the graph of the function will be:  g(x) = x - 7.

Learn more about graph function from

https://brainly.com/question/4025726

#SPJ1

Got it right on Plato/Edmentum UωU <3

What is the period of the function y=tan (4/pi (x-pi/3))
O 3 units
O4 units
O 6 units
O 8 units

Answers

Answer:

Period = 4 units

Step-by-step explanation:

Standard form of a tangent function:

[tex]f(x)=\sf A \tan(B(x+C))+D[/tex]

A = vertical stretchπ / |B| = period (distance between any two consecutive vertical asymptotes)C = phase shift (horizontal shift - positive is to the left)D = vertical shift

The tangent function has a vertical asymptote whenever cos(x) = 0

The tangent function does not have an amplitude because it has no maximum or minimum value.

Given function:

[tex]y=\tan \left(\dfrac{\pi}{4}\left(x-\dfrac{\pi}{3}\right)\right)[/tex]

Therefore:

Vertical stretch (A) = none[tex]\textsf{Period}=\dfrac{\pi}{\left|\dfrac{\pi}{4}\right|}=4[/tex]Phase shift (C) = π/3 to the rightVertical shift = none

Answer: 4 units

Step-by-step explanation:

r=
help me please thank u​

Answers

Answer:

[tex]\fbox {r = 34}[/tex]

Step-by-step explanation:

Finding IQ and RJ by tangent rule :

⇒ IQ = QK = 10

⇒ KR = RJ = 8

Finding PJ :

⇒ PJ = 32 - 8

⇒ PJ = 24

∴ But PJ = PI (by tangent rule)

⇒ PI = 24

Finding r :

⇒ PI + IQ

⇒ 24 + 10

r = 34

find the number of primes less than 190 using the principle of inclusion-exclusion.

Answers

The number of primes less than 190 using the principle of inclusion-exclusion are 42

Principle of inclusion- exclusion

The principle of inclusion-exclusion is known as a counting technique that computes the number of elements satisfying at least one of several properties and guaranteeing that the numbers are not counted twice.

Prime numbers are numbers only divisible by 1 and itself.

Prime numbers less than 190 are:

2, 3, 5, 7, 11, 13, 17, 19, 23, 29, 31, 37, 41, 43, 47, 53, 59, 61, 67, 71, 73, 79, 83, 89, 97, 101, 103, 107, 109, 113, 127, 131, 137, 139, 149, 151, 157, 163, 167, 173, 179, 181

They are 42 in number

Therefore, the number of primes less than 190 using the principle of inclusion-exclusion are 42

Learn more about prime numbers here:

https://brainly.com/question/874965

#SPJ11

If AB is parallel to CD and the slope of AB is -3, what is the slope of CD?

Answers

in order for linear functions to be parallel, the slope needs to be the same, so the slope of CD has to be -3
It should be -3 as well, if there parallel they are equal, parallel lines have equal slopes. Since the slope of AB is -3 and CD is parallel to AB then CD has a slope of -3.

pleasee help with function graphing

Answers

Answer: A. The function has two distinct real zeros.

Answer:

C. The function has four distinct real zeros.

Step-by-step explanation:

The function intersects y=0 at x=1/2, 4, and 6 twice. It has two solutions at 6 twice as it is a behavior of polynomial or rational functions

find the linear measure of arc vw on circle p, where pv = 32 inches and angle vpw = 84. use 3.14 for pie and estimate your answer to two decimal places.

Answers

The linear measure of arc vw as described in the task content is; 46.89in.

What is the linear measure of the arc vw?

It follows from the task content that the radius of the circle with center p is given as pv = 32in. and the angle subtended by the arc is; 84.

On this note, the measure of arc vw is;

= (84/360) × 2 × 3.14 × 32

= 46.89 inches.

Read more on length of an arc;

https://brainly.com/question/8402454

#SPJ1

Circle o has a circumference of 28.3 cm. What is the length of the radius r?

Answers

Circumference = 2Pi * r

28.3cm = 2pi * r ( Isolate r )
28.3cm / 2pi = r
R = 4.504

To verify work we can plug 4.504 back into the equations.

Circumference = 2pi*4.504 ~ 28.299 Which is true !

Write logb(x/y) as two logs

Answers

Answer:

[tex] log_{b}(x) - log_{b}(y) [/tex]

Is this correct? If not then whats the answer?????
(image below)

Answers

Yes that’s answer is right

Lucy is knitting a blanket and needs to buy some more yarn. at her local craft store, 2 skeins of yarn cost $7 and 8 skeins of yarn cost $28. what is the constant of proportionality in this direct variation?

Answers

Answer:

4

Step-by-step explanation:

Proportionality between skein values

8:2=4:1

Proportionality between cost values

28:7=4:1

The variation(both the skein values and cost values) has the constant of 4 ie the 1st skein value × 4= the last skein value & the 1st cost value × 4=the last cost value

35 points! Which of the following polynomials has an even degree and a positive leading coefficient?

Answers

Answer:

1

Even degree: the start and the end of the graph goes either both up or down (1 and 4)

positive leading coefficient: right side of the graph is going up (1 and 2)

Shreya and Shanice are selling pies for a
school fundraiser. Customers can buy
blueberry pies and lemon meringue pies.
Shreya sold 4 blueberry pies and 2 lemon
meringue pies for a total of $76. Shanice
sold 4 blueberry pies and 13 lemon
meringue pies for a total of $230. What is
the cost each of one blueberry pie and
one lemon meringue pie?

Answers

Answer:

$26

Step-by-step explanation:

Let blueberry pies = x and lemon pies = y.

4x + 13y = 230

4x + 2y = 76

Using elimination, we see that 11y = 154. Therefore, y = 14.

4x + 2(14) = 76

4x + 28 = 76

4x = 48

x = 12

x + y = 12 + 14 = 26.

$26 is your answer.

Answer:

Step-by-step explanation:

Let B be the price of a blueberry pie.  Let L be the price of a lemon meringue pie.

We are told that Shreya makes a total of $76 by selling 4 blueberry and 2 lemon meringue pies.  This can be made into an equation:

Shreya:  4B + 2L = $76

We also learn that Shanice is doing particularly well:

Shanice:  4B + 13L = $320

We want to determine the prices for each type of pie, B and L.  We have two equations and two unknowns.  That means we should be able to find the answers by substitution.

Rearrange Shreya's equation to isolate B:

Shreya:  4B + 2L = $76

               4B   = $76 - 2L

                 B = ($76-2L)/4

Now use this definition of B in Shanice's equation:

Shanice:  4B + 13L = $320

                4( ($76-2L)/4) + 13L = $320

                     ($76-2L) + 13L = $320

            11L = 244

                L = $22.18

Use L = $22.18 in either equation and solve for B:

4B + 2L = $76

4B + 2*($22.18) = $76

4B = $31.64

B = $7.91

---

Blueberry pies are $7.91 each.

Lemon meringue pies are $22.18 each

=========

CHECK:  Do these prices provide the correct sales for each person?

            Price($/pie)        Shreya     Shanice

Blueberry          7.91               4            4

Lemon Mer.    22.18                2           13

Blueberry Sales ($)                    31.64        31.64

Lemon Mer. Sales ($)               44.3       288.36

Total Sales                              $76        $320

YES - These prices account for each person's sales.

Calculate the AREA of the shape below. Give your answer in cm²

Answers

Answer:

[tex] \boxed{\bf 26\: cm^2} [/tex]

Step-by-step explanation:

We got two shapes in the given figure,on the right we got a rectangle and on the left we have a square.

Area of rectangle:-

[tex]\bf 5 \times 2[/tex][tex]\bf 10 \:cm^{2} [/tex]

Area of square :-

[tex]\bf 4 \times 4[/tex][tex]\bf 16 \: {cm}^{2} [/tex]

Total area :-

[tex]\bf 10 + 16[/tex][tex]\boxed{\bf 26 \: {cm}^{2} }[/tex]_________________________

Answer:

26 cm^2

Step-by-step explanation:

you have two rectangles, the bigest has the base of 6cm and the height of 4cm, the area is 6 * 4 = 24cm ^ 2, the other rectangle has the base of 6 - 4 = 2 and the height of 5 - 4 = 1, 2*1=2.

we add the dimensions and we have the area of ​​the complete figure 24 + 2 =

26 cm ^ 2

Drag each tile to the correct box. Not all tiles will be used.
Place the steps for finding in the correct order.

Answers

Answer+Step-by-step explanation:

[tex]f^{-1}\left( x\right) =\frac{1}{7} x^{2}+3 \ ,\text{where}\ x\geq 0[/tex]

Make u the subject formula
Please help

X=4uy/M​

Answers

Answer:

[tex]\frac{XM}{4y}=u[/tex]

Step-by-step explanation:

First, multiply both sides by M.

X M = 4uy

Now, divide both sides by 4y.

[tex]\frac{XM}{4y} =\frac{4uy}{4y}[/tex]

Therefore,

[tex]\frac{XM}{4y}=u[/tex]

4. John has his money in a savings
account that earns 3% interest each
year. He never takes money out of the
account. The value of his account is
described by the function
Dollars (years), or D(y).
Is D(2) < D(7) true or false?

Answers

Answer:

false

Step-by-step explanation:

so D(2) is equal to how much money he has after 2 years while D(7) is after 7 years. since he earns 3% interest each year he will have more money after 7 years than 2 years

PLEASE HELP ANSWER MY QUESTION ASAP!

Answers

[tex]\quad \huge \quad \quad \boxed{ \tt \:Answer }[/tex]

[tex]\qquad \tt \rightarrow \: a_{33} = 1\degree[/tex]

____________________________________

[tex] \large \tt Solution \: : [/tex]

In a matrix, [tex] \sf a_{ij} [/tex] represents an element in " i " th row and " j " th column.

Henceforth, element [tex] \sf a_{33} [/tex] represents element in 3rd row and 3rd column.

[tex]\qquad \tt \rightarrow \:a_{33} = 1[/tex]

Answered by : ❝ AǫᴜᴀWɪᴢ ❞

If a plane including pints p,q and r cuts through the cube

Answers

If a plane including pints p,q, and r cuts through the cube, the shape that will be gotten is a triangle.

What is a triangle?

A triangle is a polygon with three edges and three vertices. The sum of the internal angles of a triangle is equal to 180 degrees.

In this case, when a plane including pints p,q, and r cuts through the cube, the shape that will be gotten is a triangle.

The diagram is attached.

Learn more about triangles on:

https://brainly.com/question/2644832

#SPJ1

Combine the radicals 4√7 + 3√28

Answers

Answer:

10√7

Step-by-step explanation:

you can simplify the second radical and break it up into

√4 * √7 which then simplifies to 2√7 which you then multiply by 3. 4√7 + 3(2√7) which becomes 4√7 + 6√7 and then you just add the numbers Infront to get 10√7

What is the measure of

Answers

60.......................

the sum of a certain number and -3​

Answers

Answer:

[tex]\huge\boxed{\sf x - 3 }[/tex]

Step-by-step explanation:

Let the certain number be x

So,

Given condition:

Sum is to add so we'll use the sign +

= x + (-3)

We know that + and - makes - because - sign will flip the + sign and make it negative, So:

= x - 3

[tex]\rule[225]{225}{2}[/tex]

Solution: c-3

[tex]\searrow[/tex]

Detailed explanation:

[tex]\searrow[/tex]

This problem tells us to convert the verbal phrase (in words) "the sum of a certain number and -3" into an algebraic expression, which is written in numbers.

Let's start by taking a look at our problem.

"the sum of a certain number and -3". We can first write this "certain number" as one letter (any letter you want). Let it be c.

Our expression already looks a little easier: "the sum of c and -3".

Now, the word "sum" tells us to add. So we should add c and -3.

[tex]--\mapsto\boldsymbol{c+-3}[/tex]

Which is the exact same thing as [tex]\boldsymbol{c-3}[/tex].

Voila! There's our answer.

Cheers!! ^-^

______________________

Hope I helped! Best wishes.

Reach far. Aim high. Dream big.

_______________________

A piece of climbing equipment at a playground is 6 feet high and extends 4 feet horizontally. A piece of climbing
equipment at a gym is 10 feet high and extends 6 feet horizontally. Which statement best compares the slopes of the
two pieces of equipment?

Answers

Answer:

First Option i.e 5/3 > 3/2 , the slope of Climbing equipment at Gym is greater is Correct

Step-by-step explanation:

This question deals with Slope of a line.

Slope tells how vertical a line is.

The more the slope is, the more the line is vertical. When slope is zero, the line is horizontal.

To find the slope, we take the ratio of how much the line's height increases as we go forward or backward on the horizontal axis.

For first climbing equipment rise = 6 feet, run(horizontal) = 4 feet

Thus, slope =  6/4 = 3/2 ≈ 1.5

For second climbing equipment rise = 10 feet, run(horizontal) = 6 feet

Thus, slope=   10/6 = 5/3 ≈ 1.66

Thus,

Slope of first climbing equipment = 3/2 < slope of second climbing equipment = 5/3

Thus, First Option i.e 5/3 > 3/2 , the slope of Climbing equipment at Gym is greater is Correct

Learn more about slopes here:

https://brainly.com/question/9246501

#SPJ10

f(x) = 4x² + 7x-3
g(x) = 6x³ – 7x² - 5
-
Find (f + g)(x).

Answers

Answer:

(f + g)(x) = 6x³ - 3x² + 7x - 8  

Step-by-step explanation:

Another way of writing (f + g)(x) is f(x) + g(x). So, this question is asking you to combine both of the functions.

f(x) + g(x)                                          

(4x² + 7x - 3) + (6x³ - 7x² - 5)                      <----- Insert functions

-3x² + 7x - 3 + 6x³ - 5                                 <----- Combine 4x² and -7x²

-3x² + 7x - 8 + 6x³                                       <----- Combine -3 and -5

6x³ - 3x² + 7x - 8                                       <----- Rearrange

True or false: The graph of y=3x2-3 opens up.

Answers

Answer: True: The graph of y=3x²-3 opens up

Graph the parabola using the direction, vertex, focus, and axis of symmetry.

Direction: Opens Up

Vertex: (0,−3)(0,-3)

Focus: (0,−35/12)

Axis of Symmetry: x=0

Directrix: y=−37/12

X={-2, -1, 0, 1, 2}

Y={9, 0, -3, 0, 9

Step-by-step explanation:

Y = 3x + 10
Y = 2x - 8
find the substitution
Show your work please ill give brainlyest ​

Answers

Answer:

x = -18, Y = -44

Step-by-step explanation:

Since Y = 2x - 8, we can substitute that into the first equation:

2x - 8 = 3x + 10

3x - 2x = -8 - 10

x = -18

Now we plug x = -18 back into the second equation:

Y = 2(-18) - 8

Y = -36 - 8

Y = -44

please help thank you !

Answers

Answer:

6.6 cm and 14.6 cm

Step-by-step explanation:

(a)

the length of arc AB is calculated as

AB = circumference of circle × fraction of circle

     = 2πr × [tex]\frac{95}{360}[/tex]

     = 2π × 4 × [tex]\frac{95}{360}[/tex]

     = 8π × [tex]\frac{95}{360}[/tex]

     = [tex]\frac{8\pi (95)}{360}[/tex]

     ≈ 6.6 cm ( to the nearest tenth )

(b)

the perimeter (P) of sector AOB is

P = r + r + AB = 4 + 4 + 6.6 = 14.6 cm

Step-by-step explanation:

3.

the circumference of a circle is

2×pi×r

r = 4 cm

so, we know, the full circle circumference is

2×pi×4 = 8×pi = 25.13274123... cm

a.

the arc length of AB is the part of the whole circumference that corresponds to 95° out of the full 360° of a whole circle.

arc AB = 8×pi × 95/360 = pi × 95/45 = pi × 19/9 =

= 6.632251158... cm

b.

the perimeter of OAB (the "pie slice") is then the arc AB plus 2 radius lengths (from the end points on the arc to the center of the circle) :

pi × 19/9 + 2×4 = pi×19/9 + 8 = 14.632251158... cm

Aiden has a smart phone data plan that costs $25 per month that includes 2 GB of data, but will charge an extra $10 per GB over the included amount. How much would Aiden have to pay in a month where he used 2 GB over the limit? How much would Aiden have to pay in a month where he used went over by xx GB?

Total cost when over by 2 GB:
Total cost when over by xx GB:
need answer quick

Answers

Answer:

45 dollars for going over 2GB

Step-by-step explanation:

If he pays 25 dollars every month and he goes over 2 GB then he’ll have to pay 20 dollars extra since every GB over cost 10 dollars. The equation is 25+10+10 which equals 45.

An expression is a mixture of terms that are combined by using operations such as subtraction, addition, multiplication, and division. 

The amount when 2 GB is used over the limit in a month is $45.

What is an expression?

An expression is a mixture of terms that are combined by using operations such as subtraction, addition, multiplication, and division. 

Example: 2x + 3x2  is an expression.

We have,

Cost of data plan with 2 GB = $25

Cost of one GB = $10

The equation that can be used to find the amount to pay when using x GB over the limit in a month:

= 25 + 10x

The amount when 2 GB is used over the limit in a month:

= 25 + 10 x 2

= 25 + 20

= $45

Thus,

The amount when 2 GB is used over the limit in a month is $45.

Learn more about expressions here:

https://brainly.com/question/10978794

#SPJ2

Other Questions
An oxygen cylinder should be taken out of service and refilled when the pressure inside it is less than: When did Singapores big trash problem begin ?Please answer quickly its urgent and if you dont know please dont answer which type of energy do living things use to make sugar using cabon doixide? Las manos del chico _____ tan sucias que tuvo que lavrselas varias veces. given triangle PQR with QR = 8, PQ = 6, and m p = 90 degrees, which of the following choices is used to determine m r is when adolescents reach sexual maturity and become capable of PLEASE HELP ASAPThe file is attached Adrian, a single man who wants to buy a house in five years, read an article that recommended a down payment of 20 percent. With a large income and little debt, Adrian can afford to save a substantial amount of money every month. He is asking you for advice to help him reach his goal. It is now five years later, and Adrian has saved enough money for a 20 percent down payment on a house. He will have to borrow $135,000 in a 30-year loan with an annual interest rate of 6 percent compounded monthly. What will his monthly mortgage payment be Indigo earned a grade of 93% on her multiple choice history final that had a total of200 problems. How many problems on the final exam did Indigo answer correctly? Why was the BIOS framework relocated to flash memory from a complementary metal-oxide-semiconductor (CMOS) in later development? Scientists used alpha particles to investigate models of the atom. Figure 4 shows an alpha particle scattering experiment.Explain how scientists used the results from the experiment to : disprove the plum pudding model give evidence for the new nuclear model You work for a lending institution and are tasked with whether or not to approve a home loan, using the standard 28/36 ratio. the loan application is for $230,000. you see that the applicant has an annual salary of $83,000. the applicant also has a car payment of $315, a student loan of $140 and a boat loan of $96. how likely are you to approve the loan? a. very likely; recurring debt is considerably less than what is allowed. b. somewhat likely; recurring debt is very close to what is allowed. c. not likely; recurring debt is higher than what is allowed. d. there is not enough information given to determine the answer. An ongoing promotion at a department store gives customers 20\%20%20, percent off the portion of their bill that is over \$100$100dollar sign, 100. Ruby's total bill at the department store after the promotion has been applied is \$250$250dollar sign, 250. If xxx represents the amount of money Ruby would have spent on the same purchase at the department store without the promotion, which of the following equations best models the situation? How in the world do you play this tablature? I just started learning guitar and this is one of the exercises, but I don't know what the 0 is, where the letters come in, what in the world the f over the tab means, I just need it all explained and how to play, thank you! A car with a mass of 1,324 kilograms, traveling at a speed of 20 meters/second, crashes into a wall and stops. What is the kinetic energy after this perfectly inelastic collision? answer both questions please [tex]2^{x^2-5x}=\frac{1}{64}[/tex] the nurse is assessing vital signs in a patient with a permanent pacemaker. what should the nurse document about the pacemaker? Please help and fast for brainliestThe graph shows the number of gallons of white paint that were mixed with gallons of green paint in various different ratios:Draw the graph on a grid. The title is Mixing Paint. The horizontal axis label is Green Paint in gallons. The scale is 0 to 24 in increments of 3. The vertical axis label is White Paint in gallons. The scale is 0 to 72 in increments of 9. Plot points at the ordered pairs 3,9 and 6,18 and 9,27.The number of gallons of white paint mixed with 1 gallon of green paint is ______. The graph of a function f(x) is shown. What is the value of x where f(x) = -5? And f(0)?